LSAT and Law School Admissions Forum

Get expert LSAT preparation and law school admissions advice from PowerScore Test Preparation.

 Administrator
PowerScore Staff
  • PowerScore Staff
  • Posts: 8916
  • Joined: Feb 02, 2011
|
#23612
Complete Question Explanation

Strengthen—PR. The correct answer choice is (C)

The author of this stimulus presents the conclusion in the last sentence: it is more important for household products to be carefully tested than prescription pharmaceuticals. This conclusion is based on two premises:

Premise: The more people who use a product, the greater the chance of potential harm.

Premise: More people use household products than use prescription pharmaceuticals.

The question stem requires that we find the principle which most helps to justify the author's conclusion that it is more important for household products to be carefully tested than prescription pharmaceuticals. It will likely involve more common usage as a justification for greater care.

Answer choice (A): This stimulus does not deal with the question of whether or not a product should be carefully tested, but rather with relative degrees of careful testing.

Answer choice (B): Like answer choice (A) above, this answer choice deals with careful testing in general, whereas the author of the stimulus is concerned with which of two product types should be more carefully tested.

Answer choice (C): This is the correct answer choice, and the one which basically restates our prephrase from above: if this principle is valid, then this makes the author's case, which concludes that it is more important to test household products.

Answer choice (D): This answer choice deals with frequency of dosage, while the stimulus deals with the number of people who use a give product, so this answer choice fails to strengthen the author's argument.

Answer choice (E): The general distinction between medical and non-medical products is not discussed by the author, who is interested in the specific case of household product versus prescription medication.
 ShannonOh22
  • Posts: 70
  • Joined: Aug 15, 2019
|
#67946
I would love a little help on this one! I incorrectly chose E, and though I read through the explanation previously provided on the forum, I am still confused.

The author makes a direct reference to non-medical products ("household maintenance products such as cleaning agents and lawn chemicals"), and also specifically mentions medicine ("prescription medicines") in the stimulus. The argument itself is making a strange claim, and one that most people would disagree with. If a person is ingesting a medicine regularly, certainly it is more important that the medicine is tested than a household cleaner that one may or may not ever use. Answer choice E states exactly this, with the caveat "UNLESS more people's health would be at risk from the nonmedical product than from the medicine". This caveat ties it into the argument being made, and JUSTIFIES the author's strange claim that a household cleaner should be tested over a medicine someone is taking (presumably to help cure an ailment therefore the antithesis of a health hazard).

The explanation provided on the forum for E being incorrect is that the author "did not discuss medical versus non-medical products" - but that's exactly what the entire stimulus is about. And the question is looking for us to justify the strange argument, which E does, while C - though I can understand it being correct as well - merely restates the first sentence of the stimulus, it does not attempt to justify anything.

Please help!! Thanks!
 Jeremy Press
PowerScore Staff
  • PowerScore Staff
  • Posts: 1000
  • Joined: Jun 12, 2017
|
#67970
Hi Shannon,

On a Strengthen-Principle question such as this one, the ideal principle in the answer choices will be one that makes the premises sufficient to lead to the conclusion. Why? Because the best kind of support we can have for any argument is the kind that says the premises are enough (i.e. sufficient) to show that the conclusion is true. Answer choice C is that kind of principle. There is the following implied conditional relationship in answer choice C: IF there are more people whose health might be at risk from the regular use of a particular product, THEN it is more important for that product to be carefully tested to ensure its safety. Notice that the sufficient condition of that principle (the IF condition) corresponds (as you noted in your response) to the premises of the argument. We know from the premises that there are more people whose health might be at risk from the regular use of the household products identified. The necessary condition of that principle (the THEN condition) corresponds to the conclusion of the argument: in the stimulus, the author concludes it's more important for household products to be carefully tested. That's just about perfect when it comes to justifying the conclusion. Answer choice C means the premises in the stimulus are enough (sufficient) to show that the conclusion is necessarily true.

One major structural problem with answer choice E is that it reverses that logic. What I don't want the principle (the correct answer) to do in a Strengthen-Principle question is to say that the premises are NECESSARY for the conclusion to be true. We already know that. In other words, it's implicit in any argument that the premises are a necessary foundation for the conclusion. So, a principle that states that a set of premises are necessary for a conclusion is not providing any additional help to the argument. Answer choice E is that kind of principle. Remember, "unless" is a necessary condition indicator, and in answer choice E, "unless" is modifying what we see in the premises of the argument (the idea of a greater number whose health is at risk). That means answer choice E is treating the premises as necessary for the conclusion. That won't get us any closer to justifying the conclusion than we already were.

There is one other minor problem with answer choice E: the use of the term "would." We don't know from the stimulus that more people's health would (certainly) be at risk from household products. We know that more people's health is "potentially" (possibly) at risk. That possibility idea is mirrored in answer choice C, which states "[t]he more people whose health might be at risk." That makes C a better match as well.

I hope this helps!

Jeremy
 ShannonOh22
  • Posts: 70
  • Joined: Aug 15, 2019
|
#67974
Jeremy, thank you SO much!!! You are the first person who has actually broken it down for me in a way that makes sense - I was struggling with the WHY behind the correct answers, and you explained it beautifully. I honestly cannot thank you enough, you have changed the course of my studying from here on out. This is why PowerScore is the best investment one can make when buying books and study aids for the LSAT - well done, sir!! :)
 Jeremy Press
PowerScore Staff
  • PowerScore Staff
  • Posts: 1000
  • Joined: Jun 12, 2017
|
#68003
You're very welcome, Shannon--we love this stuff, and most of all we love when we can help people understand and apply it successfully! Best of luck in your continued studying, and keep working hard!
User avatar
 mhlsat
  • Posts: 5
  • Joined: Feb 02, 2023
|
#99139
For answer choice A, had it said "Whether or not it is MORE important for a given product to be carefully tested depends mainly on the number of people who regularly use that product", would that have made it correct? Because how it's originally written deals with whether its important or NOT for a given product to be tested, not the degree of importance. thank you!!
 Luke Haqq
PowerScore Staff
  • PowerScore Staff
  • Posts: 739
  • Joined: Apr 26, 2012
|
#99175
Hi mhlsat!

The conclusion on this question is stated in terms of what is "more important": "Therefore, it is even more important for such household products to be carefully tested to ensure their safety than it is for prescription medicines to be so tested."

To your question, adding just the word "more" wouldn't make answer choice (A) correct. Most pertinently, this is because the question asks for the answer choice that "most helps to justify." Compared to the other answer choices, this would be answer choice (C).

If we took away answer choice (C) and the other answer choices, looking only at (A) in isolation, adding that word still wouldn't make answer choice (A) correct. This is because of the ambiguity of the second half of the answer choice, starting at "depends": "Whether or not it is [more] important for a given product to be carefully tested depends mainly on the number of people who regularly use that product." This doesn't tells us in what way it depends. Does it depend on more people? On less people? That needs to be more explicit, as it is in answer choice (C).

Get the most out of your LSAT Prep Plus subscription.

Analyze and track your performance with our Testing and Analytics Package.